What are the 3 types of sides in a right triangle?

Answers

Answer 1

The 3 types of sides in a right triangle are hypotenuse, base, and perpendicular.

A triangle is a form of a two-dimensional polygon with three edges and vertices. It is divided on the basis of sides as a Scalene triangle, Isosceles triangle, and Equilateral triangle. It is also divided on the basis of angles as acute, obtuse, and right-angle triangles.

A right-angle triangle has one angle equal to 90° and the other two angle sums up to be 90°.  The angle formed by the base and perpendicular is 90°.

The hypotenuse is the longest side of the triangle and it is opposite of the right angle. The relation between the three sides is Hypotenuse∧2 = Perpendicular∧2 + Base∧2. this relation is also called Pythagoras theorem.

To know more about the right triangle visit:

https://brainly.com/question/1248322

#SPJ4


Related Questions

please help with this

Answers

Answer:

a) x = 1, x = -7

b) (-3, -48)

c) x = -3

d) upwards

Step-by-step explanation:

Given quadratic equation:

[tex]y=3(x-1)(x+7)[/tex]

Part (a)

Intercept form of a quadratic equation:  

 [tex]y=a(x-p)(x-q)[/tex]

where:

p and q are the x-interceptsa is some constant

Comparing the formula with the given equation:

⇒ p = 1

⇒ q = -7

Therefore, the x-intercepts of the given equation are x = 1 and x = -7.

Part (b)

The midpoint between the two x-intercepts is the x-coordinate of the vertex.

[tex]\implies \textsf{Midpoint}=\dfrac{x_2+x_1}{2}=\dfrac{1+(-7)}{2}=-3[/tex]

Therefore, the x coordinate of the vertex is -3.

To find the y-coordinate of the vertex, substitute this into the given equation:

[tex]\implies y=3(-3-1)(-3+7)=-48[/tex]

Therefore, the coordinates of the vertex are (-3, -48).

Part (c)

The x-coordinate of the vertex is the axis of symmetry.

Therefore, the axis of symmetry is x = -3.

Part (d)

If the leading coefficient of a quadratic equation is positive, the parabola opens upwards.

If the leading coefficient of a quadratic equation is negative, the parabola opens downwards.

From inspection of the given equation, the leading coefficient is 3.

Therefore, the parabola opens upwards.

Learn more about quadratic equations here:

https://brainly.com/question/27997764

Given the following results from a spinner game after spinning 20 times.
What is the experimental probability of landing on red?

Answers

The experimental probability of landing on red is 7/20 or 0.35 or 35%

How to determine the experimental probability of landing on red?

The complete question is added as an attachment

From the question, we have the following outcomes:

Red = 7

Green = 5

Blue = 5

Yellow = 3

The total number of spins is

Total = Red + Green + Blue + Yellow

This gives

Total = 7 + 5 + 5 + 3

Evaluate the sum

Total = 20

The experimental probability of landing on red is the calculated as:

P(Red) = Red/Total

Substitute the known values in the above equation

P(Red) = 7/20

Evaluate

P(Red) = 0.35

Express as percentage

P(Red) = 35%

Hence, the experimental probability of landing on red is 7/20 or 0.35 or 35%

Read more about probability at:

https://brainly.com/question/25870256

#SPJ1

Jamie and owen joined a monthly walking challenge at their youth center. jamie has already walked 120 minutes and plans to walk 20 minutes per day. owen plans to walk 30 minutes per day. let x represent the number of days and y represent the total number of minutes walked during the challenge. which system of equations can be used to find the number of days it will take for owen and jamie to have walked the same number of minutes? a system of equations. y equals 20 x plus 120. y equals 30 x. a system of equations. y equals 20 x. y equals 30 x plus 120. a system of equations. y equals 120 x plus 20. y equals 30 x. a system of equations. y equals 120 x. y equals 30 x plus 20.

Answers

Answer:

The system of equation y=120+20x and y=30x

Step-by-step explanation:

Let the number of days be x and y be the total number minutes

Jamie has walked 120 mins. She will walk at a rate of 20 mins per day.

y=120+20x

Owen walks 30 mins a day. So

y = 30x

To learn more about Linear Equations visit https://brainly.com/question/1884491

#SPJ4

Hat is the area of the shaded face of the cylinder is 22m give your answer to the nearest whole number and give the correct units

Answers

The area of the shaded face of the cylinder is 1,520 mm².

How to find the area of the shaded area?

We can see that the shaded area of the cylinder is circular in shape.

This means that we can find the area of the shaded area by finding the area of the circle.

The radius of the circle is given as 22 mm.

The formula for finding the area of a circle is given as:

Area = πr²

= 3.14 × 22 × 22

= 1,519.76

≈ 1520 mm²

Therefore, we have found the area of the shaded face of the cylinder to be 1,520 mm².

Learn more about the area of a circle here: https://brainly.com/question/14068861

#SPJ4

Disclaimer: The question was incomplete, the complete question is attached below.

9x/4y=1 and y= 18, what is the value of x​

Answers

Answer:

x = 8

Step-by-step explanation:

[tex]\frac{9x}{4y}=1[/tex]

[tex]\frac{9x}{4*18} =1[/tex]

9x = 72

x = 8

Kim's softball team was playing in the championship game. When there were 444 innings left, the team was losing by a score of 171717 to 666 runs. In the last 444 innings, her team scored the same number of runs per inning, and the other team did not score any more runs. Kim's team won with the most runs.
Write an inequality to determine the number of runs per inning, ppp, Kim's team could have scored.
Find the minimum whole number of runs per inning Kim's team could have scored.

Answers

The inequality to determine the number of runs per inning, p Kim's team could have scored is; 4r + 6 > 17

How to write an Inequality?

Let r represent the number of runs per inning. Thus for 4 innings, we have 4r.

The team already has 6 runs. Now add the additional runs to this to get;

4r + 6

The team wants to score more than the other team, this means they need more than 17 and so the inequality required is;

4r + 6 > 17

Subtract 6 from each side to get;

4r + 6 - 6 > 17 - 6

4r > 11

Divide both sides by 4 to get:

r > 2.75

Approximating to a whole number gives;

r > 3

Read more about writing inequalities at; https://brainly.com/question/25275758

#SPJ1

Answer:

6+4p> 17
3

Step-by-step explanation:

Graph the image of this triangle after a dilation with a scale factor of 2 centered at the origin. Use the polygon tool to graph the triangle.

Answers

The triangle is illustrated below.

How to explain the triangle?

The first thing you should find are the new vertices:

(x, y) ---> (2x, 2y) ---> (x', y')

(0, 0) ---> (2 (0), 2 (0)) ---> (0, 0)

(-4, 4) ---> (2 (-4), 2 (4)) ---> (-8, 8)

(-4, -2) ---> (2 (-4), 2 (-2)) ---> (-8, -4)

Then, you must join the ordered pairs and graph the new triangle.

See the attached graph.

Learn more about triangles on:

https://brainly.com/question/1058720

#SPJ1

When multiplying or dividing measured quantities, what determines the number of significant figures in the result?.

Answers

The quantity with the fewest number of significant figures.

The least number of significant figures is used to calculate the product's or quotient's number of significant figures when multiplying or dividing measurable numbers. While 5.25 has three major figures, 3.5 only has two.

What is significant figures?

Significant figures are the number of digits in a value, often a measurement, that contribute to the degree of accuracy of the value. We start counting significant figures at the first non-zero digit. Calculate the number of significant figures for an assortment of numbers.

When multiplying two measured values the number of significant figures should be equal to the?

For multiplication or division, the rule is to count the number of significant figures in each number being multiplied or divided and then limit the significant figures in the answer to the lowest count. An example is as follows: The final answer, limited to four significant figures, is 4,094.

Learn more about significant figures

brainly.com/question/14804345

#SPJ4

A dindercal container close at boat and have a radius 7cm and hight 6cm find total surface area of the container what is the volume of this container

Answers

A) The total surface area of the given cylindrical container is 571.76 cm² and B) The volume of the given cylindrical container is 923.62 cm³.

What are the formulae for the total surface and volume of the cylinder?

Consider a cylinder with height 'h', and radius 'r'.

The total surface area of the cylinder(TSA) = 2πr (r + h) square units

and

The volume of the cylinder V = πr²h cubic units.

Calculation:

It is given that,

A cylindrical container closed on both ends has a radius r = 7 cm and a height h = 6 cm.

So,

TSA = 2πr (r + h)

       = 2× π × 7 × (7 + 6)

       = 14 × π × 13

      = 571.76 cm²

and

Volume = πr²h

             = π × 7² × 6

             = π × 49 × 6

             = 923.62 cm³

Therefore, the total surface area and volume of the cylindrical container are 571.76 cm² and 923.62 cm³ respectively.

Learn more about the cylinder and its surface area and volume here:

https://brainly.com/question/27795300

#SPJ4

Which represents the solution set to the inequality 5.1(3 2.2x) > –14.25 – 6(1.7x 4)? x < –2.5 x > 2.5 (–2.5, [infinity]) (–[infinity], 2.5)

Answers

The solution set to the inequality 5.1(3 + 2.2x) > –14.25 – 6(1.7x + 4) gives x > -2.5

What is an equation?

An equation is an expression that shows the relationship between two numbers and variables.

An independent variable is a variable that does not depend on any other variable for its value whereas a dependent variable is a variable that depend on any other variable for its value.

Inequality shows the non equal comparison of two or more numbers and variables.

Given the inequality:

5.1(3 + 2.2x) > –14.25 – 6(1.7x + 4)

Simplifying:

15.3 + 11.22x > –14.25 – 10.2x - 24

21.42x > -53.55

x > -2.5

The solution set to the inequality 5.1(3 + 2.2x) > –14.25 – 6(1.7x + 4) gives x > -2.5

Find out more on equation at: https://brainly.com/question/2972832

#SPJ1

Vector u has initial points at (21,12) and it’s terminal point at (19,-8). Vector v has a direction opposite that of u. Who’s magnitude is five times the magnitude of v. Which is the correct form of vector v expressed as a linear combination of the unit vectors i and j?

Answers

The correct form of vector v expressed as a linear combination of the unit vectors i and j is [tex]\vec v = 10\,\hat{i} + 100\,\hat{j}[/tex].

What is the value of a vector with respect to another vector?

First, we need to determine the value of the vector u by subtracting two vectors whose initial points are at the origin:

[tex]\vec u = (19\,\hat{i} - 8\,\hat{j}) - (21\,\hat{i} + 12\,\hat{j})[/tex]

[tex]\vec u = - 2\,\hat{i} - 20\,\hat{j}[/tex]     (1)

According to the statement, vector v is antiparallel to vector u and its magnitude is five times as the magnitude of vector v, which means that (1) must be multiplied by two scalars:

[tex]\vec v = - 1 \,\cdot \, 5\cdot \vec u[/tex]      (2)

Please notice that antiparallelism is represented by the scalar - 1, whereas the dilation is represented by the scalar 5.

[tex]\vec v = 10\,\hat{i} + 100\,\hat{j}[/tex]

The correct form of vector v expressed as a linear combination of the unit vectors i and j is [tex]\vec v = 10\,\hat{i} + 100\,\hat{j}[/tex].

Remark

The statement presents typing mistakes, correct form is shown below:

Vector u has initial points at (21, 12) and its terminal point at (19, - 8). Vector v has a direction opposite that of u, whose magnitud is five times the magnitud of v. Which is the correct form of vector v expressed as a linear combination of the unit vectors i and j?

To learn more on vectors: https://brainly.com/question/13322477

#SPJ1

What is the explicit formula for this sequence?
-9,-3,3,9,15

Answers

Answer: A

Step-by-step explanation:

Normally in choice questions, I would choose to substitute the values into the formulas but I think it is better if I explain to you.

In a sequence, if the numbers increase by d each time and the first number in the sequence is [tex]a_{1}[/tex], then the explicit formula is [tex]a_{1}[/tex] + (n - 1)d when n is the order of the number in the sequence, like n = 1 for the first number, n = 2 for the second number and so on. Using this, we get that the answer to this problem is [tex]a_{n}[/tex] = -9 + (n - 1)6 so the answer is A

-6-5
(-5-4)
(-2,2)
2-
4
--2-1₁
4-
2
4 9
(0, -3)
X
What is the equation of the line that is parallel to the
given line and passes through the point (-2, 2)?
Oy=x+4
Oy = x + ¹/2
Oy = -5x + 4
Oy=-5x + ¹2

Answers

The equation of the line that is parallel to the given line and passes through the point (-2,2) is:

[tex]y = \frac{x}{5} + \frac{12}{5}[/tex]

What is a linear function?

A linear function is modeled by:

y = mx + b

In which:

m is the slope, which is the rate of change, that is, by how much y changes when x changes by 1.b is the y-intercept, which is the value of y when x = 0, and can also be interpreted as the initial value of the function.

When two lines are parallel, they have the same slope. In this problem, the given line passes through (-5,-4) and (0,-3), hence the slope is:

m = (-3 - (-4))/(0 - (-5)) = 1/5.

Hence the equation is:

[tex]y = \frac{x}{5} + b[/tex]

When x = -2, y = 2, then:

[tex]y = \frac{x}{5} + b[/tex]

[tex]2 = \frac{-2}{5} + b[/tex]

[tex]b = \frac{12}{5}[/tex]

Hence:

[tex]y = \frac{x}{5} + \frac{12}{5}[/tex]

More can be learned about linear equations at https://brainly.com/question/24808124

#SPJ1

3x-2y=10 find the y intercept and the x intercept

Answers

Answer:

(0, -5)      (10/3, 0)

Step-by-step explanation:

First, convert the given equation into the slope-intercept form, y = mx + b, where m represents the slope and b represents the y-intercept.

3x - 2y = 10

2y = 3x - 10

y = 3/2x - 5

The y-intercept is -5.

Next, to find the x-intercept, let y = 0 and solve for x. X-intercept is the point where the graph intersects the x-axis, so the y-coordinate of x-intercepts are always 0.

3x - 2*0 = 10

3x - 0 = 10

3x = 10

x = 10/3

X-INTERCEPT

Plug y=0 into the equation and solve the resulting equation 3x=10 for x.

The x-intercept:

[tex]\left(\frac{10}{3},0\right)\approx \left(3.33333333333333,0\right)[/tex]

Y-INTERCEPT

Plug x=0 into the equation and solve the resulting equation −2y=10 for y.

The y-intercept:

[tex]\left(0, -5\right)[/tex]

Please help with this

Answers

Answer: Circle lines

Step-by-step explanation:

Hopefully the attached image helps, it is a diagram of all the labeled lines on a circle excluding the radius (BG or GE)

Which of these is a zero of the polynomials p(y) = 3y^3 - 16 y - 8? * - 8 0 2 - 2

Answers

The zero of polynomials is -2. The polynomials is p(y) = [tex]3y^{3} - 16 y - 8[/tex].

According to the question,

The polynomials is p(y) = [tex]3y^{3} - 16 y - 8[/tex]. In order to find the zero of polynomials only if we substitute the value and polynomials become zero.

p(-2) = [tex]3(-2)^{3} - 16 (-2) - 8[/tex]

= 3(8) + 32 -8

= 0

Hence, the zero of polynomials is -2. The polynomials is p(y) = [tex]3y^{3} - 16 y - 8[/tex].

Learn more about polynomials here

https://brainly.com/question/1720316

#SPJ4

PLEASE HELP! Solve each of the following by using the general quadratic formula.

Answers

Using the quadratic formula, the solutions are:

a) [tex]x = \frac{3 \pm \sqrt{41}}{4}[/tex]

b) [tex]x = 1 \pm 2i[/tex]

What is a quadratic function?

A quadratic function is given according to the following rule:

[tex]y = ax^2 + bx + c[/tex]

The solutions are:

[tex]x_1 = \frac{-b + \sqrt{\Delta}}{2a}[/tex]

[tex]x_2 = \frac{-b - \sqrt{\Delta}}{2a}[/tex]

In which:

[tex]\Delta = b^2 - 4ac[/tex]

Item a:

The coefficients are a = 2, b = -3, c = -4, hence:

[tex]\Delta = (-3)^2 - 4(2)(-4) = 41[/tex][tex]x_1 = \frac{3 + \sqrt{41}}{4}[/tex][tex]x_2 = \frac{3 - \sqrt{41}}{4}[/tex]

Item b:

The coefficients are a = 1, b = 2, c = 2, hence:

[tex]\Delta = (2)^2 - 4(1)(2) = -4[/tex][tex]x_1 = \frac{2 + \sqrt{-4}}{2} = 1 + 2i[/tex][tex]x_2 = \frac{2 - \sqrt{-4}}{2} = 1 - 2i[/tex]

More can be learned about quadratic equations at https://brainly.com/question/24737967

#SPJ1

Can someone help with this problem please.
A sheet of paper, 12 inches by 18 inches, is folded so that 2 opposite corners touch, as shown in the figures below. What is the area, in square inches, of the shaded triangle formed as the result of the overlap. (Please look at the picture!)

Answers

The area of the shaded triangle formed as the result of the overlap is = 62.35 inches ²

Calculation of the equilateral triangle

After folding the rectangle with length of 12 inches and width of 18 inches, an equilateral triangle was formed.

An equilateral triangle is a type of triangle where by all the three sides are equal.

To determine the value of one of the sides, CB or CD is used because the folding didn't affect these sides.

Using the formula for the area of an equilateral triangle,

A = √¾ a²

a= 12 inches

A = √¾ ×12²

A = 62.35 inches ²

Learn more about triangle here:

https://brainly.com/question/1058720

#SPJ1

Answer:

Its 78!!!!

Solution:

1/2(12 * 18 - 5 * 12) = 78

Causes of variation that can be identified and eliminated are called what?

Answers

The causes of variation that can be identified and eliminated are called; Assignable Causes.

What are the causes of Variation?

There are two primary causes of variation in the quality of a product or process. These two primary causes are called;

Common causes.Assignable causes.

Now, Common causes of variation are defined as random causes that we cannot identify. However, Assignable causes of variation are those that can be identified and eliminated.

Read more about Variation Causes at; https://brainly.com/question/14926046

#SPJ1

Find the indicated term of the geometric sequence.
5th term of 1,
1 /4,1/16,…

Answers

Answer:

[tex]\frac{1}{256}[/tex]

Step-by-step explanation:

So generally a geometric sequence can be defined as: [tex]a_n=a_1(r)^{n-1}[/tex] which is the explicit form. The r, is what each previous term is being multiplied by to get the next value which is evident in the recursive form: [tex]a_n = r(a_{n-1})\\[/tex]. Knowing this we can take two values which are "next" to each other to find what r is. In this case I'll just is 1 and 1/4, given these two values we know that: [tex]\frac{1}{4} = 1 * r[/tex], 1*r is just r... so what each term is being multiplied by is 1/4. So let's plug the values into the explicit formula: [tex]a_n=(\frac{1}{4})^{n-1}[/tex] (I didn't put an a_1 value in front, since it's just 1... so it's a bit redundant). Anyways using this formula we simply plug in 5 as n into the equation to find the 5th term: [tex]a_5 = (\frac{1}{4})^{5-1} = (\frac{1}{4})^4 = \frac{1^4}{4^4} = \frac{1}{256}[/tex]

Which equations are true for x = –2 and x = 2? Select two options x2 – 4 = 0 x2 = –4 3x2 + 12 = 0 4x2 = 16 2(x – 2)2 = 0

Answers

The equations which are true for the values of x given are; x²-4 =0 and 4x²= 16.

Which equations are true for the values of x?

The values of x given according to the task content are; x = –2 and x = 2.

By considering equation; x²-4 =0

x² = 4

x = ±2

Also, by considering 4x²= 16;

x² = 16/4 = 4

x = ±4.

Read more on square root;

https://brainly.com/question/3617398

#SPJ1

Are the triangles similar? If so, state the similarity and the postulate or theorem that justifies your answer.

Answers

Answer: The triangles are not similar.

Step-by-step explanation:

Two figures are similar simply if they have the same shape, but not necessarily the same size. In a more mathematical sense, similar figures have the same angle measures and proportionate side lengths.

For reference, side lengths are proportionate when the ratios between corresponding/matching sides are the same.

Since we do not have any angles, we will try using the SSS Similarity Theorem, which states that if all three sides of both triangles are in proportion, then the triangles are similar. We will first list all the side lengths and match corresponding ones.

[tex]\triangle ABC \rightarrow 18,20,32\\ \triangle UVT \rightarrow 14,15,24[/tex]

Since we matched corresponding sides, we can now check whether they have the same ratio).

[tex]\frac{18}{14}=\frac{20}{15}=\frac{32}{24}[/tex]

[tex]\frac{9}{7}=\frac{4}{3}=\frac{4}{3}[/tex]

Not all of the side lengths have the same ratio, so these triangles aren't similar. you must multiply 14 by 9/7 to get to 18, but you need to multiply the other two sides by 4/3 to get to their

An auditorium with 30 rows of seats has 10 seats in the first row. Each successive row has one more seat than the previous row. If students taking an exam are permitted to sit in any row, but not next to another student in that row, what is the maximum number of students that can be seated for an exam

Answers

Answer:

Step-by-step explanation:

380 rows in total

bcs

use sense

David drives from his home to the airport to catch a flight. He drives 35 miles in the first hour, but realizes that he will be 1 hour late if he continues at this speed. He increases his speed by 15 miles per hour for the rest of the way to the airport and arrives 30 minutes early. How many miles is the airport from his home

Answers

The distance between David's home and the airport is 210 miles.

Distance traveled after 1 hour

let the distance traveled after first 1 hour = y

distance/speed = time

after 1 hour his speed = (35 + 15) = 50 mph

total time after 1 hour and 30 mins = 1.5 hr

y/50 + 1.5 = y/35

multiply through by350

7y + 525 = 10y

3y = 525

y = 525/3

y = 175 miles

In the first 1 hour, at 35 mph, distance covered = 35 miles

Distance from his home to airport = 175 miles + 35 miles = 210 miles

Thus, the distance between David's home and the airport is 210 miles.

Learn more about distance here: https://brainly.com/question/2854969

#SPJ1

Help help help help help just need answer

Answers

Answer:

25 miles

Step-by-step explanation:

2 inches represents 10 miles ( divide both parts by 2 )

1 inch represents 5 miles , then

5 inches represents 5 × 5 = 25 miles

A math club is researching a golf tournament fund-raiser. it will cost $1,000 to host the tournament. if it rains, the club will lose the investment. if it is sunny, it is expected that the club will collect $4,500 from the participants. if the chance of rain is 20%, what is the expected value for the tournament?

Answers

The expected value of tournament is $2600.

According to the statement

we have given that the tournament will cost $1000 and if it is sunny, it is expected that the club will collect $4,500 from the participants. And we have to find the expected value of a tournament.

So, we know that the

Cost of hosting the tournament : $1,000

If it rains, the club will lose the investment: $ -1,000

If it is sunny, the club will collect $4,500 ;

From all this the profit they will get

profit: 4,500 - 1000 = 3,500

And

There is a chance of 20% then

The club will loss the investment is

-1000*20 = -200

And

Effect on their profit if there is a rain then

3500*80 = 2800

then

The expected value of tournament is 2800-200=2600.

So,The expected value of tournament is $2600.

Learn more about Expected value here https://brainly.com/question/15858152

#SPJ4

Answer:

C

Step-by-step explanation:

on edge

The sum of a positive number and 56 times its reciprocal is equal to its cube. What is the number?

Answers

Answer:

2×sqrt(2) = 2.828427125...

Step-by-step explanation:

x = "a positive number"

x + 56/x = x³

x² + 56 = x⁴

x⁴ - x² - 56 = 0

(x² + 7)(x² - 8) = 0

this is true only for

x² = -7

or

x² = 8

x² = -7 would mean that x is an imaginary number. so, that is not a valid solution here.

x² = 8 means

x = sqrt(8) = sqrt(4×2) = 2×sqrt(2) = 2.828427125...

What criteria might CBR researchers use to select and specify which units will be sampled as well as the sampling frame and methods to be used

Answers

The criteria which CBR researchers might use to select and specify which units will be sampled as well as the sampling frame and methods to be used is the population.

What is Population?

This is defined as the total number of organisms found in a particular place at any given point in time and varies based on the number of variables present. In some cases, head counts are done to ascertain the exact value of individuals present.

The probability sampling method is ideal for a large population and involves random selection of the organisms so as to find out the statistical features of the group being used such as the range, mean etc. This tool is very important in the area of giving estimates for  ascertain type of activity or job.

Read more about Population here https://brainly.com/question/25896797

#SPJ1

A linear function on a coordinate plane. A line passing through (1, 4), (minus 2, minus 2), intersects the y- axis at 2 units and intersects the x-axis at (minus 1, 0) to form shaded portions on the left side of the line. Which of the following inequalities is graphed on the coordinate plane?

Answers

The inequality passing through (1, 4), (-2, -2), intersecting the y-axis at 2 units and intersects the x-axis at (-1, 0) to form shaded portions on the left side is y > 2x+2

What is an equation?

An equation is an expression that shows the relationship between two or more variables and numbers.

An inequality shows the non equal comparison of two or more numbers and variables.

The inequality passing through (1, 4), (-2, -2), intersecting the y-axis at 2 units and intersects the x-axis at (-1, 0) to form shaded portions on the left side is y > 2x+2

Find out more on equation at: https://brainly.com/question/2972832

#SPJ1

You buy a car for $25,000. It depreciates at the rate of 23% per year.
a. Write an exponential equation to model the value of the car.
What will the car be worth in 10 years?
round to nearest cent

Answers

The worth of the car in 10 years is $1,831.67 using an exponential equation approach.

What is an exponential equation?

An exponential equation is the one with exponents such X^3(the 3 is the exponent)

The exponential equation required here is the one where the future value would be lower than current value because the car reduces in value year-in-year-out.

FV=PV*(1-r)^N

FV=future worth of the car

PV=today's value=$25,000

r=depreciation rate=-23%

N=number of years=10

The fact that r is negative means the car is depreciating not appreciating.

FV=$25,000*(1-23%)^10

FV=$1,831.67

Find further explanation on exponential equation below in the link:

https://brainly.com/question/11832081

#SPJ1

Other Questions
product-focused creative visuals achieves business results more effciently. true false New standards for services with non-compilation reports specify which of the following (select all that apply):A. Financial statements must be prepared using US GAAP.B. Pages of the report must specify no assurance is provided.C. An engagement letter is not required.D. Independence does not need to be determined. if the partial sum with three terms is used to approximate the value of the convergent series n=3[infinity](1)n 1n2n, what is the alternating series error bound? 3.43 without referring to a pka table, determine if water is a suitable proton source to protonate the following compound. explain why or why not. How do block oriented i/o devices and stream oriented i/o devices differ? give an example of each type of device A loan is being repaid with 25 annual payments of 300 each. With the 10th payment, the borrower pays an extra 1,000, and then repays the balance over 10 years with a revised annual payment. The effective rate of interest is 8%. Calculate the amount of the revised annual payment. PLSSSSSS HELP PLS!!!!!Booker T. Washington wrote about education in 1899. The Supreme Court's decision inBrown vs. Board of Education was made in 1954. Based on the information in thepassages, compare the state of education for black children at these two points inAmerican history. Use details from both passages to support your answer 5. explain when donated materials should be recognized as a contribution and as an expense by a not-for-profit organization. Which of the following best describes the means by which most of those people leaving death row since 1976 have left?a. Most were executed.b. Most received clemency.c. Most died from causes other than execution.d. Most had their sentence or conviction overturned by an appeals or higher court. A die is selected at random from an urn that contains two six-sided dice. Die number 1 has three faces with the number 3, while one face each has the numbers 1, 2, and 4. Die number 2 has three faces with the number 2, while one face each has the numbers 1, 3, and 4. The first five rolls of the die yielded the numbers 1,3,3,2, and 4, in that order. Determine the probability that the selected die was die number 2. What is the ph of a solution containing .12mol/l nh4cl and .03mol/l naoh? The pipeline plunge is reflected across thex-axis. what are the coordinates of its newlocation? electrolysis of an nacl solution with a current of 2.02 a for a period of 203 s produced 59.3 ml of cl2 at 652 mmhg and 27 c . calculate the value of the faraday from these data.2.00 A * 200 s = 400CN = P1V1RTN= (650/760) (.0596L)/ (.0821 L*ATM/Mol*K ) (300K)N=.00207mol Cl.00207 * 2 = .004139 Cl2400C /.004139 mol = 96638 Faraday The process of evaluating relevant information that changes operating income under alternative courses of action iscalled:A. cost-benefit analysisB. incremental analysisC. opportunity cost analysisD. CVP analysis Describe a walk along the number line that (a) is unbounded, and (b) visits zero an infinite number of times. Does a series corresponding to this walk converge? An ideal gas undergoes a spontaneous expansion at constant temperature. During this process, its entropy __________Select the correct answer:a) decreases.b) remains unchanged.c) increases.d) cannot be predicted from the data given Explain how delta T would be affected if a greater amount of surrounding solvent (water) is used, assuming the mass of salt remains constant? b. Explain how q_reaction would be affected if a greater amount of surrounding solvent (water) is used? Explain. If the following enthalpies are known: A + 2B rightarrow 2C + D delta H = -95 kJ B + X rightarrow C delta H = +50kJ What is delta H for the following reaction? A rightarrow 2X + D Brodifacoun is a rat poison with a LD50 = 0.35 mg/kg. It is mixed into rat bate (50mg/kg). How much rat bait (in kg) would be needed to reach the LD50 for a 129 lb muskrat? evidence that upper paleolithic people may have been more adapted to their environments than neanderthals includes which of the following? Companies should disclose all of the following in interim reports excepta. changes in accounting principles.b. post-balance-sheet events.c. seasonal revenue, cost, or expenses.d. basic and diluted earnings per share.